Akademisyenler öncülüğünde matematik/fizik/bilgisayar bilimleri soru cevap platformu

Answers posted by sonelektrikbukucu

351
answers
33
best answers
0 votes
cevaplandı 31 Ocak 2016
$3$ ve $k$ araliklarinda polinom negatifse ikisi de denklemin koku olmali. Kokler carpimi $15$ oldug...
0 votes
cevaplandı 29 Ocak 2016
$DİLİM=53836$ $SİCİL=93238$ $TABAK=07475$ $MİLAT=63870$ $METAL=61078$ $M+A=6+7=13$
0 votes
cevaplandı 28 Ocak 2016
Eger her dolap pozitif tam bolenleri sayisinca acilip kapaniyorsa pozitif tam bolenlerinin sayisi te...
1 vote
cevaplandı 26 Ocak 2016
$y'=\frac{e^{\sqrt{x}}}{2{\sqrt{x}}}+\sqrt{x}.\frac{e^{\sqrt{x}}}{2{\sqrt{x}}}=\frac{e^{\sqrt{x}}}{2...
1 vote
cevaplandı 26 Ocak 2016
$lim_{x\rightarrow0} \frac{1+x.sinx-cos2x}{sin^2x}=lim_{x\rightarrow0} \frac{1-cos2x}{sin^2x}+lim_...
0 votes
cevaplandı 25 Ocak 2016
$15x+8=2(7x+3)+x+2$ $7x+3=7(x+2)-11$ Burada $11$ sayısı asal olduğundan $x+2=-11$ olmalıdır
0 votes
cevaplandı 25 Ocak 2016
$3.15^{2x-3}=3.5^{2x-3}.3^{2x-3}=5^{2x-1}$ olduguna gore duzenlersek $3^{2x-2}=5^2$ buluruz. $\frac{...
0 votes
cevaplandı 23 Ocak 2016
$X=\frac{sin^2 20-cos^2 20}{1-sin40}=\frac{(sin20-cos20)(sin20+cos20)}{sin^2 20+cos^2 20-2.sin20.c...
0 votes
cevaplandı 23 Ocak 2016
Eğer kareleri köşeden çıkartacak olursak çevre değişmez. Fakat eğer ortadan kesip çıkartacak olurs
0 votes
cevaplandı 23 Ocak 2016
Soru 1 için $2^p-1$ sayısının asal olmaması için bir $q$ asalına tam bölünmesi gerekir yani $2^p
0 votes
cevaplandı 23 Ocak 2016
Soruyu hallettiğimize göre cevabı yazalım. $(x^2+2x+8-4\sqrt{3})(x^2-6x+16-4\sqrt{3})=((x+1)^2+(2-
0 votes
cevaplandı 18 Ocak 2016
$\sqrt{8.11.9.10+1}=\sqrt{88.90+1}=\sqrt{(89-1)(89+1)+1}=\sqrt{89^2-1+1}=89$
0 votes
cevaplandı 18 Ocak 2016
Çubukların ilk durumdaki dışta kalan kısımlarına $k,\ 2k,\ 3k$ ve ikinci durumda dışta kalan kısımla
0 votes
cevaplandı 17 Ocak 2016
$x$ yerine $x-(-2)$ ve $y$ yerine $y-2$ yazarsak $2x^2+5x^2+8x-20y-2=2(x+2)^2+5(y-2)^2-30=0$ denklem...
0 votes
cevaplandı 16 Ocak 2016
$2013$ ve $1000$ aralarinda asal oldugundan Euler teoremi kullanirsak $1000=2^3.5^3$ oldugundan $2...
0 votes
cevaplandı 14 Ocak 2016
$2^{23}$ sayisini $9$ ve $11$ modunda incelersek $2^{23}=5 (mod 9)$ ve $2^{23}=8 (mod 11)$ olur. $14...
0 votes
cevaplandı 11 Ekim 2015
$-sin\theta+i.cos\theta=cos(90+\theta)+i.sin(90+\theta)$ ve $|cis(90+\theta)|=1$ olduğuna göre ikinc
0 votes
cevaplandı 11 Ekim 2015
Bayağı kafamı karıştırdı, uğraştırdı ama çözdüm galiba :) $20000000=2.10^7=4^4.5^7$ olduğuna g
0 votes
cevaplandı 9 Eylül 2015
$x+y+z=1$ ise $x+y=1-z$ olmalıdır. $xyz=xy+yz+zx$ koşulu sağlanıyorsa $xyz-xy=xy(z-1)=z(x+y)$ ifad
0 votes
cevaplandı 28 Temmuz 2015
$2^3.4^3...64^3=(2^3)^{32}(1^3.2^3...32^3)=2^{96}(32!)^3$ esitligini elde ettikten sonra $32!$ ifa...
20,217 soru
21,750 cevap
73,347 yorum
1,968,655 kullanıcı